(bobadilla) 11 x09 capital budgeting.doc

March 10, 2018 | Author: Karen Labasan | Category: Internal Rate Of Return, Net Present Value, Capital Budgeting, Depreciation, Present Value
Share Embed Donate


Short Description

Download (bobadilla) 11 x09 capital budgeting.doc...

Description

Capital Budgeting

MODULE 9 Risk & return 6. The higher the risk element in a project, the A. more attractive the investment is. B. higher the net present value is. C. higher the cost of capital is. D. higher the discount rate is.

CAPITAL BUDGETING THEORIES: Basic Concepts Decision Making Process 2. The first step in the decision-making process is to A. determine and evaluate possible courses of action. B. identify the problem and assign responsibility. C. make a decision. D. review results of the decision.

9. Cost of capital is the A. amount the company must pay for its plant assets. B. dividends a company must pay on its equity securities. C. cost the company must incur to obtain its capital resources. D. cost the company is charged by investment bankers who handle the issuance of equity or long-term debt securities.

Strategic planning 39. Strategic planning is the process of deciding on an organization’ A. minor programs and the approximate resources to be devoted to them B. major programs and the approximate resources to be devoted to them C. minor programs prior to consideration of resources that might be needed D. major programs prior to consideration of resources that might be needed

14. How should the following projects be listed in order of increasing risk? A. New venture, replacement, expansion. B. Replacement, new venture, expansion. C. Replacement, expansion, new venture. D. Expansion, replacement, new venture.

Capital budgeting defined 1. The long-term planning process for making and financing investments that affect a company’s financial results over a number of years is referred to as A. capital budgeting C. master budgeting B. strategic planning D. long-range planning

41. Problems associated with justifying investments in high-tech projects often include discount rates that are too A. low and time horizons that are too long B. high and time horizons that are too long C. high and time horizons that are too short D. low and time horizons that are too short

3. Capital budgeting is the process A. used in sell or process further decisions. B. of determining how much capital stock to issue C. of making capital expenditure decisions D. of eliminating unprofitable product line

60. In evaluating high-tech projects, A. only tangible benefits should be considered. B. only intangible benefits should be considered. C. both tangible and intangible benefits should be considered. D. neither tangible nor intangible benefits should be considered.

5. A capital investment decision is essentially a decision to: A. exchange current assets for current liabilities. B. exchange current cash outflows for the promise of receiving future cash inflows. C. exchange current cash flow from operating activities for future cash inflows from investing activities. D. exchange current cash inflows for future cash outflows.

Types of capital projects 4. A project that when accepted or rejected will not affect the cash flows of another project. A. Independent projects C. Mutually exclusive projects B. Dependent projects D. Both b and c 227

Capital Budgeting

recovered A. at the end of the project’s life B. in the first year of the project’s life C. evenly through the project’s life D. when the company goes out of businessA

Capital budgeting process 7. The normal methods of analyzing investments A. cannot be used by not-for-profit entities. B. do not apply if the project will not produce revenues. C. cannot be used if the company plans to finance the project with funds already available internally. D. require forecasts of cash flows expected from the project.

32. XYZ Co. is adopting just-in-time principles. When evaluating an investment project that would reduce inventory, how should XYZ treat the reduction? A. Ignore it. B. Decrease the cost of the investment and decrease cash flows at the end of the project’s life. C. Decrease the cost of the investment. D. Decrease the cost of the investment and increase the cash flow at the end of the project’s life.

Investments Sale of old asset 38. When disposing of an old asset and replacing it with a new one, tax effect on A. gain on sale of the old asset reduces the basis of the new asset B. gain on sale of the old asset increases the basis of the new asset C. loss on sale of the old asset reduces the basis of the new asset D. b and c

Relevant cash flows 72. Which of the following represents the biggest challenge in the decision to purchase new equipment? A. Estimating employee training for the new project. B. Estimating cash flows for the future. C. Estimating transportation costs of the new equipment. D. Estimating maintenance costs for the new equipment.

Working capital 18. A major difference between an investment in working capital and one in depreciable assets is that A. an investment in working capital is never returned, while most depreciable assets have some residual value. B. an investment in working capital is returned in full at the end of a project’s life, while an investment in depreciable assets has no residual value. C. an investment in working capital is not tax-deductible when made, nor taxable when returned, while an investment in depreciable assets does allow tax deductions. D. because an investment in working capital is usually returned in full at the end of the project’s life, it is ignored in computing the amount of the investment required for the project.

51. When a firm has the opportunity to add a project that will utilize factory capacity that is currently not being used, which costs should be used to determine if the added project should be undertaken? A. Opportunity costs C. Net present costs B. Historical costs D. Incremental costs 11. The only future costs that are relevant to deciding whether to accept an investment are those that will A. be different if the project is accepted rather than rejected. B. be saved if the project is accepted rather than rejected. C. be deductible for tax purposes. D. affect net income in the period that they are incurred.

30. The proper treatment of an investment in receivables and inventory is to A. ignore it B. add it to the required investment in fixed assets C. add it to the required investment in fixed assets and subtract it from the annual cash flows D. add it to the investment in fixed assets and add the present value of the recovery to the present value of the annual cash flows

Cash inflow 66. Which of the following is not a typical cash inflow in capital investment decisions?

31. In connection with a capital budgeting project, an investment in working capital is normally 228

Capital Budgeting

A. Incremental revenues B. Cost reductions

C. Salvage value D. Additional working capital

from all of the following sources except: A. debt financing B. cost savings C. salvage value D. reduction in the amount of working capital

Out-of-pocket costs 45. Which of the following is a cost that requires a future outlay of cash that is which relevant for future decision-making? A. Opportunity cost C. Sunk costs B. Out-of-pocket cost D. Relevant benefits

10. If Helena Company expects to get a one-year bank loan to help cover the initial financing of one of its capital projects, the analysis of the project should A. offset the loan against any investment in inventory or receivables required by the project. B. show the loan as an increase in the investment. C. show the loan as a cash outflow in the second year of the project’s life. D. ignore the loan

Depreciation & Tax 22. If there were no income taxes, A. depreciation would be ignored in capital budgeting. B. the NPV method would not work. C. income would be discounted instead of cash flow. D. all potential investments would be desirable.

Sunk cost 29. In deciding whether to replace a machine, which of the following is NOT a sunk cost? A. The expected resale price of the existing machine. B. The book value of the existing machine. C. The original cost of the existing machine. D. The depreciated cost of the existing machine.

21. Relevant cash flows for net present value (NPV) models include all of the following except A. outflows to purchase new equipment B. depreciation expense on the newly acquired piece of equipment C. reductions in operating cash flows as a result of using the new equipment. D. cash outflows related to purchasing additional inventories for another retail store.

Accounting rate of return 54. The primary advantages of the average rate of return method are its ease of computation and the fact that: A. It is especially useful to managers whose primary concern is liquidity B. There is less possibility of loss from changes in economic conditions and obsolescence when the commitment is short-term C. It emphasizes the amount of income earned over the life of the proposal D. Rankings of proposals are necessary

55. When evaluating depreciation methods, managers who are concerned about capital investment decisions will: A. choose straight line depreciation so there is minimum impact on the decision. B. use units of production so more depreciation expense will be allocated to the later years. C. use accelerated methods to have as much depreciation in the early years of an asset’s life. D. choice of depreciation method has no impact on the capital investment decision.

Nondiscounted cash flow method Payback method 36. There are several capital budgeting decision models that do not use discounted cash flows. What is the name of the simple technique that calculates the total time it will take to recover, using cash inflows from operations, the amount of cash invested in a project? A. Recovery period C. External rate of return B. Payback model D. Accounting rate of return

70. The tax consequences should be considered under which circumstances when making capital investment decisions? A. Positive net income C. Depreciation B. Disposal of an asset D. All of the above Irrelevant cash flows Loan financing 43. In addition to incremental revenues, cash inflows from capital investments can be generated

34. The technique most concerned with liquidity is 229

Capital Budgeting

A. B. C. D.

Payback method. Net present value technique. Internal rate of return. book rate of return.

D. Net present value and internal rate of return Net present value 69. Discounted cash flow analysis is used in which of the following techniques? A. Net present value C. Cost of capital B. Payback period D. All of the above

73. Which of the following is a potential use of the payback method? A. Help managers control the risks of estimating cash flows B. Help minimize the impact of the investment on liquidity C. Help control the risk of obsolescence D. All of the answers are correct

8. The primary capital budgeting method that uses discounted cash flow techniques is the A. net present value method. B. cash payback technique. C. annual rate of return method. D. profitability index method.

47. The cash payback technique: A. should be used as a final screening tool. B. can be the only basis for the capital budgeting decision. C. is relatively easy to compute and understand. D. considers the expected profitability of a project.

20. The net present value (NPV) model can be used to evaluate and rank two or more proposed projects. The approach that computes the total impact on cash flows for each option and then converts these total cash flows to their present values is called the A. differential approach C. contribution approach B. incremental approach. D. total project approach.

33. Which of the following is NOT a defect of the payback method? A. It ignores cash flows because it uses net income. B. It ignores profitability. C. It ignores the present values of cash flows. D. It ignores the pattern of cash flows beyond the payback period.

40. The discount rate commonly used in present value calculations is the A. treasury bill rate B. weighted average return on assets adjusted for risk C. risk free rate plus inflation rate D. shareholders’ expected return on equity

48. The payback method, as a capital budgeting technique, assumes that all intermediate cash inflows are reinvested to yield a return equal to: A. Zero C. The Discount Rate B. The Time-Adjusted-Rate-of-Return D. The Cost-of-Capital 52. Which of the following capital budgeting methods is the least theoretically correct? A. payback method C. internal rate of return B. net present value D. none of the above

44. Which is true of the net present value method of determining the acceptability of an investment? A. The initial cost of the investment is subtracted from the present value of net cash flows B. The net cash flows are not adjusted to present value C. A negative net present value indicates the investment should be undertaken D. The net present value method requires no subjective judgments

Discounted cash flow method 49. Which of the following methods of evaluating capital investment projects incorporates the time value of money? A. Payback period, accounting rate of return, and internal rate of return B. Accounting rate of return, net present value, and internal rate of return C. Payback period and accounting rate of return

Profitability index 35. The profitability index A. does not take into account the discounted cash flows. B. Is calculated by dividing total cash flows by the initial investment. C. allows comparison of the relative desirability of projects that require differing initial investments. 230

Capital Budgeting

D. will never be greater than 1.0.

B. equal to the cost of borrowed capital. C. equal to zero. D. lower than the company’s cutoff rate return.

Internal rate of return 56. According to the reinvestment rate assumption, which method of capital budgeting assumes cash flows are reinvested at the project’s rate of return? A. payback period C. internal rate of return B. net present value D. none of the above

27. The relationship between payback period and IRR is that A. a payback period of less than one-half the life of a project will yield an IRR lower than the target rate. B. the payback period is the present value factor for the IRR. C. a project whose payback period does not meet the company’s cutoff rate for payback will not meet the company’s criterion for IRR. D. none of the above.

62. The rate of interest that produces a zero net present value when a project’s discounted cash operating advantage is netted against its discounted net investment is the: A. Cost of capital C. Cutoff rate B. Discount rate D. Internal rate of return

67. When comparing NPV and IRR, which is not true? A. With NPV, the discount rate can be adjusted to take into account increased risk and the uncertainty of cash flows B. With IRR, cash flows can be adjusted to account for risk C. NPV can be used to compare investments of various size or magnitude D. Both NPV and IRR can be used for screening decisions

57. A weakness of the internal rate of return method for screening investment projects is that it: A. Does not consider the time value of money B. Implicitly assumes that the company is able to reinvest cash flows from the project at the company’s discount rate C. Implicitly assumes that the company is able to reinvest cash flows from the project at the internal rate of return D. Fails to consider the timing of cash flows

Sensitivity analysis 13. In capital budgeting, sensitivity analysis is used A. to determine whether an investment is profitable. B. to see how a decision would be affected by changes in variables. C. to test the relationship of the IRR and NPV. D. to evaluate mutually exclusive investments.

Comprehensive 50. Which of the following methods of evaluating capital investment projects do not use a percentage as a measurement unit? A. Payback period and net present value B. Accounting rate of return and payback period C. Net present value and internal rate of return D. Internal rate of return and payback period

15. An approach that uses a number of outcome estimates to get a sense of the variability among potential returns is A. the discounted cash flow technique. B. the net present value method. C. risk analysis. D. sensitivity analysis.

Relationships among NPV, PI & IRR 24. If a company’s required rate of return is 12 percent and in using the profitability index method, a project’s index is greater than 1.0, this indicates that the project’s rate of return is A. equal to 12 percent. C. less than 12 percent. B. greater than 12 percent. D. dependent on the size of the investment.

42. Sensitivity analysis is the study of how the outcome of a decision making process A. changes as one or more of the assumptions change B. remains the same even though one or more of the assumptions change C. changes even though one or more of the assumptions do not change D. does not change as the assumptions do not change either

25. If the present value of the future cash flows for an investment equals the required investment, the IRR is A. equal to the cutoff rate. 231

Capital Budgeting

A. Investment decisions B. Screening decisions

64. Sensitivity analysis is: A. An appropriate response to uncertainty in cash flow projections B. Useful in measuring the variance of the Fisher rate C. Typically conducted in the post investment audit D. Useful to compare projects requiring vastly different levels of initial investment

C. Management decisions D. Preference decisions

Payback period 46. If a payback period for a project is greater than its expected useful life, the A. project will always be profitable. B. entire initial investment will not be recovered. C. project would only be acceptable if the company’s cost of capital was low. D. project’s return will always exceed the company’s cost of capital.

IRR = 0 58. if the internal rate of return on an investment is zero: A. its NPV is positive. B. its annual cash flows equal its required investment. C. it is generally a wise investment. D. its cash flows decrease over its life.

Net present value 61. An analysis of a proposal by the net present value method indicated that the present value of future cash inflows exceeded the amount to be invested. Which of the following statements best describes the results of this analysis? A. The proposal is desirable and the rate of return expected from the proposal exceeds the minimum rate used for the analysis B. The proposal is desirable and the rate of return expected from the proposal is less than the minimum rate used for the analysis C. The proposal is undesirable and the rate of return expected from the proposal is less than the minimum rate used for the analysis D. The proposal is undesirable and the rate of return expected from the proposal exceeds the minimum rate used for the analysis

Change in NPV 59. Which of the following would decrease the net present value of a project? A. A decrease in the income tax rate B. A decrease in the initial investment C. An increase in the useful life of the project D. An increase in the discount rate Effect of change in cost of capital 26. All other things being equal, as cost of capital increases A. more capital projects will probably be acceptable. B. fewer capital projects will probably be acceptable. C. the number of capital projects that are acceptable will change, but the direction of the change is not determinable just by knowing the direction of the change in cost of capital. D. the company will probably want to borrow money rather than issue stock.

63. NPV indicates a project is deemed desirable (acceptable) when the NPV is A. greater than or equal to zero B. less than zero C. greater than or equal to the risk-adjusted cost of capital D. less than or equal to the risk-adjusted cost of capital

Effect of change in residual value 23. Assuming that a project has already been evaluated using the following techniques, the evaluation under which technique is least likely to be affected by an increase in the estimated residual value of the project? A. Payback Period. C. Net Present Value. B. Internal Rate of Return. D. Profitability Index.

Internal rate of return 12. If Arbitrary Company wants to use IRR to evaluate long-term decisions and to establish a cutoff rate of return, it must be sure that the cutoff rate is A. at least equal to its cost of capital. B. at least equal to the rate used by similar companies. C. greater than the IRR on projects accepted in the past. D. greater than the current book rate of return.

Decision rules – independent projects 68. What type of decision involves deciding if an investment meets a predetermined standard?

NPV & IRR 232

Capital Budgeting

19. The NPV and IRR methods give A. the same decision (accept or reject) for any single investment. B. the same choice from among mutually exclusive investments. C. different rankings of projects with unequal lives. D. the same rankings of projects with different required investments.

17. A thorough evaluation of how well a project’s actual performance matches the projections made when the project was proposed is called a A. pre-audit. C. sensitivity analysis. B. post-audit. D. risk analysis. 37. A follow-up evaluation of a capital project is performed to see that investment expenditures are proceeding on time and on budget, to compare actual cash flows with those originally predicted, and to evaluate continuation of the project. This follow-up is called a A. postaudit. C. management audit B. performance evaluation D. project review

Decision rule – mutually exclusive projects 71. Mutually exclusive projects are those that: A. if accepted, preclude the acceptance of competing projects. B. if accepted, can have a negative effect on the company’s profit. C. if accepted, can also lead to the acceptance of a competing project. D. require all managers to consider.

65. Companies use post audits to: A. chastise managers whose project does not exceed projections. B. prove to managers that they should have accepted projects they previously rejected. C. have the managers revise poorly performing projects so the projects will have larger return in the future. D. provide feedback that enables managers to improve the accuracy of the projections of future cash flows, thereby maximizing the quality of the firm’s capital investments.

28. In choosing from among mutually exclusive investments the manager should normally select the one with the highest A. Net present value. C. Profitability index. B. Internal rate return. D. Book rate of return. 53. Why do the NPV method and the IRR method sometimes produce different rankings of mutually exclusive investment projects? A. The NPV method does not assume reinvestment of cash flows while the IRR method assumes the cash flows will be reinvested at the internal rate of return. B. The NPV method assumes a reinvestment rate equal to the discount rate while the IRR method assumes a reinvestment rate equal to the internal rate of return. C. The IRR method does not assume reinvestment of the cash flows while the NPV assumes the reinvestment rate is equal to the discount rate. D. The NPV method assumes a reinvestment rate equal to the bank loan interest rate while the IRR method assumes a reinvestment rate equal to the discount rate.

PROBLEMS: Net Investment i . Bruell Company is considering to replace its old equipment with a new one. The old equipment had a net book value of P100,000, 4 remaining useful life with P25,000 depreciation each year. The old equipment can be sold at P80,000. The new equipment costs P160,000, have a 4-year life. Cash savings on operating expenses before 40% taxes amount to P50,000 per year. What is the amount of investment in the new equipment? A. P160,000 C. P 80,000 B. P 72,000 D. P 68,000

Post-audit 16. Post-audit of capital projects A. is usually conclusive. B. is done using different evaluation techniques than were used in making the original capital budgeting decision. C. provides a formal mechanism by which the company can determine whether existing projects should be supported or terminated. D. all of the above.

Operating Cash Flow Cash Flow Before tax ii . Taal Company is considering the purchase of a machine that promises to reduce operating costs by equal amounts every year of its 6-year useful life. The machine will cost P840,000 and has no salvage value. The machine has a 20% internal rate of return. Taal Company is subject to 40% income tax rate. The present value of 1 for 6 periods at 20% is 3.326, and at the end of 6 periods is 0.3349. The approximate annual cash savings before tax is closest to: 233

Capital Budgeting

A. P252,555 B. P112,555

C. P187,592 D. P327,592

Third year 180,000 Fourth year 120,000 Fifth year 100,000 Maleen will use the sum-of-the-years-digits’ method to depreciate the new machine as follows: First year P150,000 Second year 120,000 Third year 90,000 Fourth year 60,000 Fifth year 30,000 The present value of 1 for 5 periods at 12 percent is 3.60478. The present values of 1 at 12 percent at end of each period are: End of: Period 1 0.89280 Period 2 0.79719 Period 3 0.71178 Period 4 0.63552 Period 5 0.56743 Had Maleen used straight-line method of depreciation instead of declining method, what is the difference in net present value provided by the machine at a discount rate of 12 percent? A. Increase of P 9,750 C. Decrease of P24,376 B. Decrease of P 9,750 D. Increase of P24,376

Increase in Annual Income Tax iii . Mayon Company is considering replacing its old machine with a new and more efficient one. The old machine has book value of P100,000, a remaining useful life of 4 years, and annual straight-line depreciation of P25,000. The existing machine has a current market value of P80,000. The replacement machine would cost P160,000, have a 4-year life, and will save P50,000 per year in cash operating costs. If the replacement machine would be depreciated using the straight-line method and the tax rate is 40%, what should be the increase in annual income taxes? A. P14,000 C. P40,000 B. P28,000 D. P 4,000 Depreciation & Taxes iv . Prime Consulting, Inc. operates consulting offices in Manila, Olongapo, and Cebu. The firm is presently considering an investment in a new mainframe computer and communication software. The computer would cost P6 million and have an expected life of 8 years. For tax purposes, the computer can be depreciated using either straight-line method or Sum-of-theYears’-Digits (SYD) method over five years. No salvage value is recognized in computing depreciation expense and no salvage value is expected at the end of the life of the equipment. The company’s cost of capital is 10 percent and its tax rate is 40 percent. The present value of annuity of 1 for 5 periods is 3.791 and for 8 periods is 5.335. The present values of 1 end of each period are: 1 0.9091 5 0.6209 2 0.8264 6 0.5645 3 0.6513 7 0.5132 4 0.6830 8 0.4665 The present value of the net advantage of using SYD method of depreciation with a five-year life instead of straight-line method of depreciating the equipment is: A. P 86,224 C. P215,560 B. P115,168 D. P287,893 v

.

Accounting rate of return Based on initial investment vi . A piece of labor saving equipment that Marubeni Electronics Company could use to reduce costs in one of its plants in Angeles City has just come onto the market. Relevant data relating to the equipment follow: Purchase cost of the equipment P432,000 Annual cost savings that will be provided by the equipment 90,000 Life of the equipment 12 years What is the simple rate of return to be provided by the equipment? A. Between 15% and 18%. C. 20.83%. B. 25.00%. D. 12.50%.

For P450,000, Maleen Corporation purchased a new machine with an estimated useful life of five years with no salvage value. The machine is expected to produce cash flow from operations, net of 40 percent income taxes, as follows: First year P160,000 Second year 140,000

Based on average investment vii . The BIBO Company has made an investment in video and recording equipment that costs P106,700. The equipment is expected to generate cash inflows of P20,000 per year. How 234

Capital Budgeting

many years will the equipment have to be used to provide the company with a 10 percent average accounting rate of return on its investment? A. 7.28 years C. 9.05 years B. 5.55 years D. 4.75 years viii

A. P300,000 B. P550,000

CFAT xii . The Hills Company, a calendar company, purchased a new machine for P280,000 on January 1. Depreciation for tax purposes will be P35,000 annually for eight years. The accounting (book value) rate of return (ARR) is expected to be 15% on the initial increase in required investment. On the assumption of a uniform cash inflow, this investment is expected to provide annual cash flow from operations, net of income taxes, of A. P35,000 C. P42,000 B. P40,250 D. P77,000

. Show Company is negotiating to purchase an equipment that would cost P200,000, with the expectation that P40,000 per year could be saved in after-tax cash operating costs if the equipment were acquired. The equipment’s estimated useful life is 10 years, with no salvage value, and would be depreciated by the straight-line method. Show Company’s minimum desired rate of return is 12 percent. The present value of an annuity of 1 at 12 percent for 10 periods is 5.65. The present value of 1 due in 10 periods, at 12 percent, is 0.322. The average accrual accounting rate of return (ARR) during the first year of asset’s use is: A. 20.0 percent C. 10.0 percent B. 10.5 percent D. 40.0 percent

ix

.

Payback Period xiii . If an asset costs P35,000 and is expected to have a P5,000 salvage value at the end of its tenyear life, and generates annual net cash inflows of P5,000 each year, the cash payback period is A. 8 years C. 6 years B. 7 years D. 5 years

An asset was purchased for P66,000. The asset is expected to last for 6 years and will have a salvage value of P16,000. The company expects the income before tax to be P7,200 and the tax rate applicable to the company is 30%. What is the average return on investment (accounting rate of return)? A. 17.6% C. 10.9% B. 7.6% D. 12.3%

xiv

. Consider a project that requires cash outflow of P50,000 with a life of eight years and a salvage value of P5,000. Annual before-tax cash inflow amounts to P10,000 assuming a tax rate of 30% and a required rate of return of 8%. Salvage value is ignored in computing depreciation. The project has a payback period of A. 5.0 years C. 6.0 years B. 5.6 years D. 6.6 years

Net Investment x . The Makabayan Company is planning to purchase a new machine which it will depreciate, for book purposes, on a straight-line basis over a ten-year period with no salvage value and a full year’s depreciation taken in the year of acquisition. The new machine is expected to produce cash flows from operations, net of income taxes, of P66,000 a year in each of the next ten years. The accounting (book value) rate of return on the initial investment is expected to be 12 percent. How much will the new machine cost? A. P300,000 C. P550,000 B. P660,000 D. P792,000 xi

.

C. P660,000 D. P792,000

xv

. The following incomplete information is provided for an investment decision. Discount Discounted Cumulative Cash Year Cash Flow Factor (10%) Cash Flows Flows 0 P(450,000) 1.000 P(450,000) P(450,000) 1 280,000 .909 254,520 2 210,000 .826 3 140,000 .751 Using break-even time (BET) analysis, when will the investment be recovered? A. In 2.73 years C. At the end of year 2 B. Longer than three years D. In 2.21 years

The Fields Company is planning to purchase a new machine which it will depreciate, for book purposes, on a straight-line basis over a ten-year period with no salvage value and a full year’s depreciation taken in the year of acquisition. The new machine is expected to produce cash flow from operations, net of income taxes, of P66,000 a year in each of the next ten years. The accounting (book value) rate of return on the initial investment is expected to be 12%. How much will the new machine cost?

xvi

235

. Orlando Corporation is considering an investment in a new cheese-cutting machine to replace

Capital Budgeting

its existing cheese cutter. Information on the existing machine and the replacement machine follow: Cost of the new machine P400,000 Net annual savings in operating costs 90,000 Salvage value now of the old machine 60,000 Salvage value of the old machine in 8 years 0 Salvage value of the new machine in 8 years 50,000 Estimated life of the new machine 8 years What is the expected payback period for the new machine? A. 4.44 years C. 2.67 years B. 8.50 years D. 3.78 years xvii

A. 4.6 years B. 1.9 years xix

. The Leisure Company is considering the purchase of electronic pinball machines to place in amusement houses. The machines would cost a total of P300,000, have an eight-year useful life, and have a total salvage value of P20,000. Based on experience with other equipment, the company estimates that annual revenues and expenses associated with the machines would be as follows: Revenues form use P200,000 Less operating expenses Commissions to amusement houses P100,000 Insurance 7,000 Depreciation 35,000 Maintenance 18,000 160,000 Net income P 40,000 Ignoring the effect of income taxes, the payback period for the pinball machines would be A. 3.73 years C. 4.0 years B. 3.23 years D. 7.5 years

. For P4,500,000, Siniloan Corporation purchased a new machine with an estimated useful life of five years with no salvage value at its retirement. The machine is expected to produce cash flow from operations, net of income taxes, as follows: First year P 900,000 Second year 1,200,000 Third year 1,500,000 Fourth year 900,000 Fifth year 800,000 Siniloan will use the sum-of-the-years-digits’ method to depreciate the new machine as follows: First year P1,500,000 Second year 1,200,000 Third year 900,000 Fourth year 600,000 Fifth year 300,000 What is the payback period for the machine? A. 3 years C. 5 years B. 4 years D. 2 years

xviii

C. 3.0 years D. 2.5 years

Net Present Value xx . It is the start of the year and Agudelo Company plans to replace its old grinding equipment. The following information are made available by the management: Old New Equipment cost P70,000 P120,000 Current salvage value 14,000 Salvage value, end of useful life 5,000 16,000 Annual operating costs 44,000 32,000 Accumulated depreciation 55,300 Estimated useful life 10 years 10 years The company is not subject to tax and its cost of capital is 12%. What is the present value of all the relevant cash flows at time zero? A. (P 54,000) C. (P106,000) B. (P120,000) D. (P124,700)

. Paz Insurance Company’s management is considering an advertising program that would require an initial expenditure of P165,500 and bring in additional sales over the next five years. The cost of advertising is immediately recognized as expense. The projected additional sales revenue in Year 1 is P75,000, with associated expenses of P25,000. The additional sales revenue and expenses from the advertising program are projected to increase by 10 percent each year. Paz Insurance Company’s tax rate is 40 percent. The payback period for the advertising program is

xxi

236

. Consider a project that requires an initial cash outflow of P500,000 with a life of eight years and a salvage value of P20,000 upon its retirement. Annual cash inflow before tax amounts to P100,000 and a tax rate of 30 percent will be applicable. The required minimum rate of return for this type of investment is 8 percent. The present value of 1 and the annuity of 1,

Capital Budgeting

discounted at 8 percent for 8 periods are 0.54 and 5.747, respectively. Salvage value is ignored in computing depreciation. The net present value amounts to A. P 7,560 C. P 17,606 B. P 10,050 D. P 20,050 xxii

B. P(37,064) xxiv

. Mario Hernandez plans to buy a haymaker. It costs P175,000 and is expected to last for five years. He presently hires 6 workers at P10,000 per month for each of the three harvesting months each year. The equipment would eliminate the need for two workers. Hernandez uses straight-line depreciation and projects a salvage value of P25,000. His tax rate is 25% and opportunity cost of funds is 12.0%. The present value of 1discounted at 12 percent at the end of 5 periods is 0.56743 and the present value of an annuity of 1 for 5 periods is 3.60478. Which of the following is true? A. The present value of cash flows in year 5 is P22,710 B. NPV is P28,436 C. NPV is P15,250 D. NPV is P14,186

. Zap Manufacturing has an investment opportunity to embark on a project where yearly revenues for five years are to be P400,000 and operating costs of P104,800. The equipment costs P1 million, and straight-line depreciation will be used for book and tax purposes. No salvage value is expected at the end of the project’s life. The company has a 40 percent marginal tax rate and a 10 percent cost of capital. The equipment manufacturer has offered a delayed payment plan of P560,500 per year at the end of the first and second years. There will be no changes in working capital. The present value of annuity of 1 for 5 periods is 3.7908 at 10 percent. The present values of 1 end of each period at 10 percent are: Period 1 0.9091 Period 2 0.8264 Period 3 0.7513 Period 4 0.6830 Period 5 0.6209 The net present value if the equipment were purchased is A. P (87,977) C. P 1,922 B. P (25,310) D. P (61,094)

xxiii

D. P(29,136)

xxv

. Paz Insurance Company’s management is considering an advertising program that would require an initial expenditure of P165,500 and bring in additional sales over the next five years. The cost of advertising is immediately recognized as expense. The projected additional sales revenue in Year 1 is P75,000, with associated expenses of P25,000. The additional sales revenue and expenses from the advertising program are projected to increase by 10 percent each year. Paz Insurance Company’s tax rate is 40 percent. The present value of 1 at 10 percent, end of each period: Period Present value of 1 1. 0.90909 2. 0.82645 3. 0.75131 4. 0.68301 5. 0.62092 The net present value of the advertising program would be A. P 37,064 C. P 29,136

. Tabucol Aggregates, Inc. plans to replace one of its machines with a new efficient one. The old machine has a net book value of P120,000 with remaining economic life of 4 years. This old machine can be sold for P80,000. If the new machine were acquired, the cash operating expenses will be reduced from P240,000 to P160,000 for each of the four years, the expected economic life of the new machine. The new machine will cost Tabucol a cash payment to the dealer of P300,000. The company is subject to 32 percent tax and for this kind of investment, a marginal cost of capital of 9 percent. The present value of annuity of 1 and the present value of 1 for 4 periods using 9 percent are 3.23972 and 0.70843, respectively. The net present value to be provided by the replacement of the old machine is A. P28,493 C. P46,794 B. P15,693 D. P59,594

xxvi

237

. Zambales Mines, Inc. is contemplating the purchase of equipment to exploit a mineral deposit that is located on land to which the company has mineral rights. An engineering and cost analysis has been made, and it is expected that the following cash flows would be associated with opening and operating a mine in the area. Cost of new equipment and timbers 2,750,000 Working capital required 1,000,000 Net annual cash receipts* 1,200,000 Cost to construct new road in three years 400,000 Salvage value of equipment in 4 years 650,000 *Receipts from sales of ore, less out-of-pocket costs for salaries, utilities, insurance, etc. It is estimated that the mineral deposit would be exhausted after four years of mining. At that point, the working capital would be released for reinvestment elsewhere. The company’s

Capital Budgeting

discount rate is 20%. The net present value for the project is: A. P 454,620. B. P (79,303).

Variable cost per dozen P0.38 Total fixed costs P21,000 Estimated cash value of machines: December 31, 2006 P40,000 December 31, 2012 P 7,000 Assume all operating revenues and expenses occur at the end of the year. The net advantage in present value of the better alternative is: A. Retain Old Machine, P61,675. B. Buy New Machine, P61,675. C. Retain Old Machine, P16,345. D. Buy New Machine, P16,345.

C. P(561,553) D. P(204,688).

With inflation xxvii . By the end of December 31, 2005, Alay Foundation is considering the purchase of a copying machine for P80,000. The expected annual cash savings are expected to be P32,000 in the next four years. At the end of the four years, the machine will be discarded without any salvage value. All the cash savings are stated in number of pesos at December 31, 2006. The company expected that the inflation rate is constantly 5 percent each year. Hence, the first year’s cash inflow was adjusted for 5 percent inflation. For simplicity, all cash inflows are assumed to be at year-end. The present value at 14 % of 1 for 4 periods is 2.91371. The present value of 1 at end of each period are: Period 1 0.87719 Period 2 0.76947 Period 3 0.67497 Period 4 0.59208 Using the nominal rate of return of 14 percent, the net present value for this machine is A. P12,239 C. P13,419 B. P19,670 D. P27,936 xxviii

P0.29 P 11,000 P120,000 P 20,000

Profitability index xxix . The Pambansang Kamao Corporation has to replace its completely damaged boiler machine with a new one. The old machine has a net book value of P100,000 with zero market value; therefore it will give a tax shield, based on 35% tax rate if replaced, by P35,000. The company has a 10 percent cost of capital. Understandably, the new machine, through a uniform decrease in cash operating costs, will give a positive net present value, because this machine will provide an internal rate of return of 12 percent. The present values at 10% and 12%, respectively, are: 10% 12% Annuity of 1, 6 periods 4.35526 4.11141 1 end of 6 periods 0.56447 0.50663 If the machine were to be depreciated using straight-line method for 6 years without any salvage value, the estimated profitability index is: A. 1.20 B. 1.06 C. 1.07 D. Cannot be determined from the information

. Perpetual Foundation, Inc., a nonprofit organization, has one of its activities, the production of cookies for its snack food store. Several years ago, Perpetual Foundation, Inc. purchased a special cookie-cutting machine. As of December 31, 2006, this machine will have been used for three years. Management is considering the purchase of a newer, more efficient machine. If purchased, the new machine would be acquired on December 31, 2006. Management expects to sell 300,000 dozen cookies in each of the next six years. The selling price of the cookies is expected to average P1.15 per dozen. Perpetual Foundation, Inc. has two options: continue to operate the old machine, or sell the old machine and purchase the new machine. No trade-in was offered by the seller of the new machine. The following information has been assembled to help management decide which option is more desirable. Old Machine New Machine Original cost of machine at acquisition P80,000 P120,000 Remaining useful life as of 12/31/06 6 years 6 years Expected annual cash operating expenses:

xxx

238

. The Mejicano Company is planning to purchase a piece of equipment that will reduce annual cash expenses over its 5-year useful life by equal amounts. The company will depreciate the equipment using straight-line method of depreciation based on estimated life of 5 years without any salvage value. The company is subject to 40 percent tax. The marginal cost of capital for this acquisition is 11.055 percent. The management accountant calculated that the internal rate of return based on the estimated after-tax cash flows is 12.386 percent and a net present value of P10,000. The president, however, wants to know the profitability index before he finally decides.

Capital Budgeting

What is the profitability index for this investment? A. 1.011 C. 1.022 B. 1.034 D. 1.044

taxes, of P20,000 in each of the five years. Forest’s expected rate of return is 10%. Information on present value and future amount factors is as follows: PERIOD 1 2 3 4 5 Present value of P1 at 10% .909 .826 .751 .683 .621 Present value of an annuity of P1 at 10% .909 1.736 2.487 3.170 3.791 Future amount of P1 at 10% 1.100 1.210 1.331 1.464 1.611 Future amount of an annuity of P1 at 1.00 2.100 3.310 4.641 6.105 10%

Internal Rate of Return xxxi . Diamond Company is planning to buy a coin-operated machine costing P400,000. For book and tax purposes, this machine will be depreciated P80,000 each year for five years. Diamond estimates that this machine will yield an annual inflow, net of depreciation and income taxes, of P120,000. Diamond’s desired rate of return on its investments is 12%. At the following discount rates, the NPVs of the investment in this machine are: Discount Rate NPV 12% +P3,258 14% + 1,197 16% - 708 18% - 2,474 Diamond’s expected IRR on its investment in this machine is A. 3.25% C. 16.00% B. 12.00% D. 15.30%

How much will the machine cost?

A. P 32,220 B. P 62,100

Required unit sales xxxv . Paper Products Company is considering a new product that will sell for P100 and has a variable cost of P60. Expected volume is 20,000 units. New equipment costing P1,500,000 and having a five-year useful life and no salvage value is needed, and will be depreciated using the straight-line method. The machine has fixed cash operating costs of P200,000 per year. The firm is in the 40 percent tax bracket and has cost of capital of 12 percent. The present value of 1, end of five periods is 0.56743; present value of annuity of 1 for 5 periods is 3.60478. How many units per year the firm must sell for the investment to earn 12 percent internal rate of return? A. 17,338 C. 9,838 B. 28,897 D. 12,338

Required investment xxxii . Kipling Company has invested in a project that has an eight-year life. It is expected that the annual cash inflow from the project will be P20,000. Assuming that the project has a internal rate of return of 12%, how much was the initial investment in the project if the present value of annuity of 1 for 8 periods is 4.968 and the present value of 1 is 0.404? A. P160,000 C. P 80,800 B. P 99,360 D. P 64,640 xxxiii

C. P 75,820 D. P122,100

Required selling price xxxvi . Bughaw Products Company is considering a new product that will sell for P100 and has a variable cost of P60. Expected sales volume is 20,000 units. New equipment costing P1,500,000 with a five-year useful life and no terminal salvage value is needed. The machine will be depreciated using the straight-line method. The machine has cash operating costs of P200,000 per year. The firm is in the 40 percent tax bracket and has cost of capital of 12 percent. The present value of 1, end of five periods is 0.56743; present value of annuity of 1 for 5 periods is 3.60478. Suppose the 20,000 estimated sales volume is sound, but the price is in doubt, what is the selling price (rounded to nearest peso) needed to earn a 12 percent internal rate of return? A. P81.00 C. P70.00 B. P95.00 D. P90.00

. Katol Company invested in a machine with a useful life of six years and no salvage value. The machine was depreciated using the straight-line method. It was expected to produce annual cash inflow from operations, net of income taxes, of P6,000. The present value of an ordinary annuity of P1 for six periods at 10% is 4.355. The present value of P1 for six periods at 10% is 0.564. Assuming that Katol used a time- adjusted rate of return of 10%, what was the amount of the original investment? A. P10,640 C. P22,750 B. P29,510 D. P26,130

xxxiv

. The Forest Company is planning to invest in a machine with a useful life of five years and no salvage value. The machine is expected to produce cash flow from operations, net of income 239

Capital Budgeting

2. 3. 4. 5. 6.

Disposal value after 8 years – Zero. Estimated net annual cash inflows for each of the 8 years – P81,000. Time-adjusted internal rate of return – 14% Cost of capital of Bayan Muna – 16% The table of present values of P1 received annually for 8 years has these factors: at 14% = 4.639, at 16% = 4.344 7. Depreciation is approximately P46,970 annually. Find the required increase in annual cash inflows in order to have the time-adjusted rate of return approximately equal the cost of capital. A. P6,501 C. P4,344 B. P5,501 D. P5,871

Required CFBT xxxvii . Aloha Co. is considering the purchase of a new ocean-going vessel that could potentially reduce labor costs of its operation by a considerable margin. The new ship would cost P500,000 and would be fully depreciated by the straight-line method over 10 years. At the end of 10 years, the ship will have no value and will be sunk in some already polluted harbor. The Aloha Co.’s cost of capital is 12 percent, and its marginal tax rate is 40 percent. If the ship produces equal annual labor cost savings over its 10-year life, how much do the annual savings in labor costs need to be to generate a net present value of P0 on the project? Use the following PV: annuity of 1, 10 periods at 12% - 5.6502; end of 10th period – 0.32197. A. P 68,492 C. P114,154 B. P147,487 D. P 88,492

Required CFAT for a certain year xli . A company is considering putting up P50,000 in a three-year project. The company’s expected rate of return is 12%. The present value of P1.00 at 12% for one year is 0.893, for two years is 0.797, and for three years is 0.712. The cash flow, net of income taxes will be P18,000 (present value of P16,074) for the first year and P22,000 (present value of P17,534) for the second year. Assuming that the rate of return is exactly 12%, the cash flow, net of income taxes, for the third year would be A. P23,022 C. P10,000 B. P 7,120 D. P16,392

Required CFAT xxxviii . Prudu Company has decided to invest in some new equipment. The equipment will have a three-year life and will produce a uniform series of cash savings. The net present value of the equipment is P1,750, using a discount rate of 8 percent. The internal rate of return is 12 percent. Present values at 8% and 12% respectively: 8%: Annuity – 2.5771; end of 3 periods, 0.7938 12%: Annuity – 2,4018; end of 3 periods, 0.7118 What is the amount of annual cash inflow? A. P 9,980 C. P23,240 B. P21,342 D. P12,351

Required salvage value xlii . The Caravan Company is contemplating to purchase a machine that costs P800,000. The machine is expected to last for 5 years with a salvage value of P50,000 at the end of the fifth year. If the machine were purchased, before-tax annual cash savings on operating expenses will be realized. Caravan Company will depreciate the machine using straight-line depreciation for 5 years, with the salvage value considered in the computation. The company has a 12 percent cost of capital and is subject to 40 percent tax rate. The present values using 12 percent are: Annuity of 1 for 5 periods 3.60478 Present value of 1, end of 5 periods 0.56743 The initial analysis indicated a net present value of P7,003. You believe the estimated beforetax cash savings are fairly determined but you are in doubt of the expected salvage value of the machine. How much is the estimated salvage value required if the investment has to yield an IRR of 12 percent? A. P41,800 C. P25,100

xxxix

. An asset is purchased for P120,000. It is expected to provide an additional P28,000 of annual net cash inflows. The asset has a 10-year life and an expected salvage value of P12,000. The hurdle rate is 10%. The present value of an annuity factor of 10% for 10 years is 6.1446, and the present value of P1, discounted for 10 years at 10% is 0.3855. Given the data provided, the minimum amount of annual cash inflows that would provide the 10% time-adjusted return is approximately A. P18,776 C. P24,400 B. P26,600 D. P22,535

Required Increase in CFAT xl . The following data pertain to Julian Corp. whose management is planning to purchase a unit of equipment. 1. Economic life of equipment – 8 years. 240

Capital Budgeting

B. P24,900

D. P44,600 xlvi

Required value of intangible benefits xliii . Solidum Company is investigating the purchase of a piece of automated equipment that will save P100,000 each year in direct labor and inventory carrying costs. This equipment costs P750,000 and is expected to have a 10-year useful life with no salvage value. The company requires a minimum 15% return on all equipment purchases. Management anticipates that this equipment will provide intangible benefits such as greater flexibility and higher quality output. The PV of annuity of 1, 15% for 10 periods 5.01877 The PV of 1, end 10 period 0.24718 What peso value per year would these intangible benefits have to have in order to make the equipment an acceptable investment? A. P248,123 C. P 61,331 B. P 49,440 D. P 55,000

. Silky Products is considering two pieces of machinery. The first machine costs P50,000 more than the second machine. During the two-year life of these two alternatives, the first machine has a P155,000 more cash flow in year one and a P110,000 less cash flow in year two than the seconds machine. All cash flows occur at year-end. The present value of 1 at 15 percent end of 1 period and 2 periods are 0.86957 and, 0.75614, respectively. The present value of 1 at 8 percent end of period 1 is 0.92593, and Period 2 is 0.85734. At what discount rate would Machine 1 be equally acceptable as machine 2’s? A. 9% C. 11% B. 10% D. 12%

Decision Rule – Independent Projects xlvii . Sylvia Products is considering two types of machinery. The first machine costs P50,000 more than the second machine. During the two-year life of these two alternatives, the first machine has a P155,000 more cash flow in year one and a P110,000 less cash flow in year two than the seconds machine. All cash flows occur at year-end. The present value of 1 at 15 percent end of 1 period and 2 periods are 0.86957 and, 0.75614, respectively. The present value of 1 at 8 percent end of period 1 is 0.92593, and Period 2 is 0.85734. Which machine should be purchased if the relevant discount rates are 15 percent and 8 percent, respectively? 15% Discount 8% Discount A. Machine 1 Machine 1 B. Machine 2 Machine 2 C. Machine 1 Machine 2 D. Machine 2 Machine 1

xliv

. Altas, Inc., is considering investing in automated equipment with a ten-year useful life. Managers at Altas have estimated the cash flows associated with the tangible costs and benefits of automation, but have been unable to estimate the cash flows associated with the intangible benefits. Using the company’s 10% discount rate, the net present value of the cash flows associated with just the tangible costs and benefits is a negative P184,350. The present value of annuity of 1 at 10 percent for ten years is 6.145 while the present value of 1 is 0.386. How large would the annual net cash inflows from the intangible benefits have to be to make this a financially acceptable investment? A. P18,435. C. P35,000. B. P30,000. D. P37,236.

Indifference Point xlv . Moon Company uses a 10% discount rate and the total cost approach to capital budgeting analysis. Both alternatives are Akda Investments which has a marginal cost of capital of 12 percent is evaluating two mutually exclusive projects (X and Y), which have the following projections: PROJECT X PROJECT Y Investment P48,000 P83,225 After-tax cash inflow 12,000 15,200 Asset life 6 years 10 years The indifference point for the two projects is A. 12.64% C. 12.00% B. 16.01% D. 19.33%

Comprehensive Payback, NPV, ARR Question Nos. 71 through 73 are based on the following: Cayco Medical Center is considering purchasing an ultrasound machine for P950,000. The machine has a 10 – year life and an estimated salvage value of P55,000. Installation costs and freight charges will be P24,200 and P800, respectively. Newman uses straight-line depreciation. The medical center estimates that the machine will be used five times a week with the average charges to the patient for ultrasound of P800. There are P10 in medical supplies and P40 of technician costs for each procedure performed using the machine. The present value of an annuity of 1 for 10 years at 9% is 6.418 while the present value of 1 for 10 years at 9% is 0.42241 241

Capital Budgeting

xlviii

. The cash payback period is: A. 3.0 years B. 4.5 years

ARR, NPV, PI, Payback Questions 1 through 4 will be based on the following data: The management of Arleen Corporation is considering the purchase of a new machine costing P400,000. The company’s desired rate of return is 10%. The present value of P1 at compound interest of 10% for 1 through 5 years are 0.909, 0.826, 0.751, 0.683, and 0.621, respectively, and the present value of annuity of 1 for 5 periods at 10 percent is 3.79. In addition to the foregoing information, use the following data in determining the acceptability in this situation:

C. 5.0 years D. 6.0 years

xlix

. The project is expected to generate net present value of: A. P276,510 C. P331,510 B. P299,743 D. P253,277

l

.

Year 1 2 3 4 5

What is the accounting rate of return provided by the project? A. 20.0 percent C. 11.2 percent B. 10.6 percent D. 38.0 percent

NPV, CFAT, Maximum lost unit sales Question Nos. 75 through 77 are based on the following: Kabalikat Company has the opportunity to introduce a new product. Kabalikat expects the product to sell for P75 with variable cost per unit of P50. The annual fixed costs, excluding the amount of depreciation is P4,500,000. The company expects to sell 300,000 units. To produce the new product line, the company needs to purchase a new machine that costs P6,000,000. The new machine is expected to last for four years with a very negligible salvage value. The company has a policy of depreciating its machine for both book and tax purposes for four years. The company has a marginal cost of capital of 13.75 percent and is subject to tax rate of 40 percent. li

.

lii

.

liii

Net Cash Flow P180,000 120,000 100,000 90,000 90,000

liv

. The average rate of return for this investment is: A. 18 percent C. 58 percent B. 6 percent D. 10 percent

lv

.

The amount of annual after-tax cash flows is: A. P2,400,000 C. P 900,000 B. P3,000,000 D. P1,500,000 The machine’s net present value is: A. P2,786,100 B. P 928,500

Income from Operations P100,000 40,000 20,000 10,000 10,000

lvi

. Assuming that some of the 300,000 units that are expected as sales would be to group of customers who currently buy K-Z, another product of Kabalikat Company. This Product K-Z sells for P35 with variable cost of P20. How many units of K-Z can Kabalikat afford to lose before the purchase of the new machine becomes unattractive? A. 39,000 units C. 16,714 units B. 23,400 units D. 10,029 units

C. Negative P 99,600 D. Negative P126,800

. The present value index for this investment is: A. 0.88 C. 1.14 B. 1.45 D. 0.70

lvii

C. P1,028,900 D. P 150,270

The net present value for this investment is: A. Positive P 36,400 B. Positive P 55,200

. The cash payback period for this investment is: A. 4 years C. 20 years B. 5 years D. 3 years

Payback, NPV, ARR, IRR Use the following information for questions 67 - 70 Pillo Company is considering two capital investment proposals. Estimates regarding each project are provided below:

242

Capital Budgeting

Project MA P2000,000 10,000 50,000 5 years -0-

Initial investment Annual net income Net annual cash inflow Estimated useful life Salvage value

A. P1,875 B. P7,000

Project PA P300,000 21,000 71,000 6 years -0-

lxiii

. Annual cash flow after tax amounts to A. P 1,875 B. P 7,000 . Payback amounts to A. 5.0 years B. 5.6 years

C. 6.0 years D. 6.6 years

. Net present value amounts to A. P 756 B. P1,005

C. P1,756 D. P2,005

Present Value of an Annuity of 1

lviii

9% 3.890 4.486

10% 3.791 4.355

. The cash payback period for Project MA is A. 20 years B. 10 years

11% 3.696 4.231

lxv

12% 3.605 4.111

lxvi

C. 5 years D. 4 years

lx

.

lxi

The annual rate of return for Project MA is A. 5% B. 10%

C. P 50,000 D. P 9,205 C. 25% D. 50%

Alternative 1. The pizza shop in this location is currently selling 40,000 pizzas per year. Management is confident that sales could be increased by 75% by taking out the wall between the pizza shop and the vacant space and expanding the pizza outlet. Costs for remodeling and for new equipment would be P550,000. Management estimates that 20% of the new sales would be small pizzas, 50% would be medium pizzas, and 30% would be large pizzas. Selling prices and costs for ingredients for the three sizes of pizzas follow (per pizza):

. The internal rate of return for Project PA is closest to A. 10% C. 12% B. 11% D. none of these

Depreciation tax shield, CCFAT, Payback, NPV, IRR Question Nos. 86 through 90 are based on the following: Consider a project that requires cash outflow of P50,000 with a life of eight years and a salvage value of P2,000. Annual cash inflow amounts to P10,000 assuming a tax rate of 30% and a required rate of return of 8%. Salvage value is ignored in computing depreciation. lxii

. Internal rate of return on this project is approximatel A. 8.0% C. 9.0% B. 8.5% D. 9.5%

CFAT, NPV, IRR Questions 46 rough 51 are based on the following: Home’s Pizza’s, Inc., operates pizza shops in several cities. One of the company’s most profitable shops is located adjacent to the large CPA review center in Manila. A small bakery next to the shop has just gone out of business, and Home’s Pizzas has an opportunity to lease the vacated space for P18,000 per year under a 15-year lease. Home’s management is considering two ways in which the available space might be used.

lix

. The net present value for Project PA is A. P309,204 B. P 91,456

C. P 8,875 D. P10,000

lxiv

The company requires a 10% rate of return on all new investments.

Period 5 6

C. P8,875 D. P10,000

Small Medium Large

. Annual depreciation tax shield amounts to 243

Selling Price P 6.70 8.90 11.00

Cost of Ingredients P1.30 2.40 3.10

Capital Budgeting lxix

An additional P7,500 of working capital would be needed to carry the larger volume of business. This working capital would be released at the end of the lease term. The equipment would have a salvage value of P30,000 in 15 years, when the lease ends. Alternative 2. Home’s sales manager feels that the company needs to diversify its operations. He has suggested that an opening be cut in the wall between the pizza shop and the vacant space and that video games be placed in the space, along with a small snack bar. Costs for remodeling and for the snack bar facilities would be P290,000. The games would be leased from a large distributor of such equipment. The distributor has stated that based on the use of game centers elsewhere, Home’s could expect about 26,000 people to use the center each year and to spend an average of P5 each on the machines. In addition, it is estimated that the snack bar would provide a net cash inflow of P15,000 per year. An investment of P4,000 in working capital would be needed. This working capital investment would be released at the end of the lease term. The snack bar equipment would have a salvage value of about P12,000 in 15 years.

Rent- building space Rent- video games Salaries Utilities Insurance and other

C. P68,375 D. P12,807

. Assume that the company decides to accept alternative 2. At the end of the first year, the company finds that only 21,000 people used the game center during the year (each person spent P5 on games). Also, the snack bar provided a net cash inflow of only P13,000. In light of this information, what is the net present value for alternative 2? A. P(80,422) C. P(82,150) B. P(76,422) D. P(80,854)

lxxii

Install the Game Center P18,000 30,000 17,000 5,400 9,600

. The sales manager has suggested that an advertising program be initiated to draw another 5,000 people into the game center each year. Assuming that another 5,000 people can be attracted into the center and that the snack bar receipts increase to the level originally estimated, how much can be spent on advertising each year and still allow the game center to provide a 16% rate of return? A. P70,103.00 C. P58,953.00 B. P 4,673.53 D. P12,574.53

Net Income, CFBT, ARR, Payback Period Questions 52 through 56 are based on the following information: Pinewood Craft Company is considering the purchase of two different items of equipment, as described below: Machine A. A compacting machine has just come onto the market that would permit Pinewood Craft Company to compress sawdust into various shelving products. At present the sawdust is disposed of as a waste product. The following information is available on the machine:

. The incremental expected annual cash inflows from Alternative 1 is: A. P 90,000 C. P100,200 B. P108,000 D. P201,000

lxviii

. The net present value for Alternative 2 is: A. P21,021 B. P70,103

lxxi

The company is currently using a 16 percent minimum acceptable rate of return for its capital investment. The present value of annuity of 1 at 16 percent for 15 periods is 5.575 and end of 15 periods is 0.108. The company is not liable to pay income taxes. lxvii

C. P45,000 D. P32,500.

lxx

Home’s management is unsure which alternative to select and has asked you to help in making the decision. You have gathered the following information relating to added costs that would be incurred each year under the two alternatives: Expand the Pizza Shop P18,000 --54,000 13,200 7,800

. The net present value for Alternative 1 is: A. P48,650 B. P47,840

a. The machine would cost P420,000 and would have a 10% salvage value at the end of its 12-year useful life. The company uses straight-line depreciation and considers salvage value in computing depreciation deductions. b. The shelving products manufactured from use of the machine would generate revenues of P300,000 per year. Variable manufacturing costs would be 20% of sales. c. Fixed expenses associated with the new shelving products would be (per year):

. The incremental expected annual cash inflows from Alternative 2 is: A. P 17,000 C. P 59,600 B. P 65,000 D. P145,000

244

Capital Budgeting

advertising, P40,000; salaries, P110,000; utilities, P5,200; and insurance, P800.

A. 3.0 years B. 4.5 years

Machine B. A second machine has come onto the market that would allow Pinewood Craft Company to automate a sanding process that is now done largely by hand. The following information is available:

lxxviii

a. The new sanding machine would cost P234,000 and would have no salvage value at the end of its 13-year useful life. The company would use straight-line depreciation on the new machine. b. Several old pieces of sanding equipment that are fully depreciated would be disposed of at a scrap value of P9,000. c. The new sanding machine would provide substantial annual savings in cash operating costs. It would require an operator at an annual salary of P16,350 and P5,400 in annual maintenance costs. The current, hand-operated sanding procedure costs the company P78,000 per year in total.

C. 6.1 years. D. 5.9 years.

Net Investment, CFBT, Tax Benefits, NPV, Depreciation Tax Shield, Question Nos. 58 through 63 are based on the following: Turkey Company’s average production of valve stems over the past three years has been 80,000 units each year. Expectations are that this volume will remain constant over the next four years. Cost records indicate that unit product costs for the valve stem over the last several years have been as follows: Direct materials Direct labor Variable manufacturing overhead Fixed manufacturing overhead* Unit product cost

Pinewood Craft Company requires a simple rate of return of 15% on all equipment purchases. Also, the company will not purchase equipment unless the equipment has a payback period of 4.0 years or less. (In all the following questions, please ignore income tax effect) lxxiii

. The payback period for Machine B is: A. 4.0 years. B. 4.2 years.

C. 5.0 years D. 7.5 years

P 3.60 3.90 1.50 9.00 P18.00

*Depreciation of tools (that must now be replaced) accounts for one-third of the fixed overhead. The balance is for other fixed overhead costs of the factor that require cash expenditures.

. The expected income each year from the new shelving products (Machine A) is: A. P 52,500 C. P 84,000 B. P240,000 D. P 92,500

If the specialized tools are purchased, they will cost P2,500,000 and will have a disposal value of P100,000 at the end of their four-year useful life. Turkey Company has a 30% tax rate, and management requires a 12% after-tax return on investment. Straight-line depreciation would be used for financial reporting purposes, but for the tax purposes, the following variable depreciation each year will be used.

lxxiv

. The annual savings in cost if Machine B is purchased is A. P56,250 C. P38,250 B. P43,250 D. P21,750

lxxv

. The simple rate (%) of return for Machine A is: A. 12.5 percent C. 25.0 percent B. 20.0 percent D. 18.0 percent

lxxvi

. The simple rate of return for Machine B is: A. 16.3 percent B. 17.0 percent

lxxvii

Year 1 Year 2 Year 3 Year 4

C. 25.0 percent D. 34.0 percent

P 832,500 1,112,500 370,000 185,000

The sales representative for the manufacturer of the specialized tools has stated, “The new tools will allow direct labor and variable overhead to be reduced by P1.60 per unit.” Data from another company using identical tools and experiencing similar operating conditions, except that annual

. The payback period for Machine A is: 245

Capital Budgeting

production generally averages 100,000 units, confirms the direct labor and variable overhead cost savings. However, the other company indicates that it experienced an increase in raw material cost due to the higher quality of material that had to be used with the new tools. The other company indicates that its unit product costs have been as follows: Direct materials Direct labor Variable manufacturing overhead Fixed manufacturing overhead Unit product cost

B. P 216,000 lxxxi

P 4.50 3.00 0.80 10.80 P19.10

. The present value of the salvage value of the new tools to be received at the end of fourth year is A. P 63,552. C. P 44,486. B. P 19,065. D. P212,615.

lxxxiii

Although the old tools being used by Turkey Company are now fully depreciated, they have a salvage value of P45,000. These tools will be sold if the new tools are purchased; however if the new tools are not purchased, then the old tools will be retained as standby equipment. Turkey Company’s accounting department has confirmed that total fixed manufacturing overhead costs, other than depreciation, will not change regardless of the decision made concerning the valve stems. However, the accounting department has estimated that working capital needs will increase by P60,000 if the new tools are purchased due to the higher quality of material required in the manufacture of the valve stems. The present values of 1 at the end of each period using 12 percent are: Period 1 Period 2 Period 3 Period 4 PV of annuity of 1, 4 periods

. The present value of tax benefits expected from the use of the new machine tools is: A. P 603,333 C. P1,407,777 B. P 804,444 D. P2,011,111

lxxxii

Referring to the figures above, the production manager stated, “These numbers look great until you consider the difference in volume. Even with the reduction in labor and variable overhead cost, I’ll bet our total unit cost figure would increase to over P20 with the new tools.”

D. P1,008,000

. Using the minimum acceptable rate of return of 12 percent, the net present value of the investment in new tools is A. P108,913. C. P147,073. B. P127,979. D. P166,139.

lxxxiv

. The net advantage of the use of declining method of depreciation instead of straight-line method is A. P 33,830. C. P112,767. B. P 56,610. D. P147,731.

Net Investment, CFAT, Depreciation tax shield, NPV Question Nos. 77 through 82 are based on the following: Franzen Company manufactures three different models of paper shredders including the waste container, which serves as the base. While the shredder heads are different for all three models, the waste container is the same. The number of waste containers that Franzen will need during the next five years is estimated as follows: 2007 50,000 2008 50,000 2009 52,000 2010 55,000 2011 55,000 The equipment used to manufacture the waste container must be replaced because it is broken and cannot be repaired. The new equipment would have a purchase price of P945,000 with terms 2/10, n/30; the company’s policy is to take all purchase discounts. The freight on the equipment would be P11,000, and installation costs would total P22,900. The equipment would be purchased in December 2006 and placed into service on January 1, 2007. It would have a five-year economic

0.89286 0.79719 0.71178 0.63552 3.03735

lxxix

. The net investment in new tools amounted to: A. P1,873,300. C. P2,528,500. B. P2,515,000. D. P2,546.500.

lxxx

. How much annual cost savings will be generated if the Turkey Company purchases the new tools? A. P 128,000 C. P 936,000 246

Capital Budgeting lxxxv

life and would have the following depreciation. The equipment is expected to have a salvage value of P12,000 at the end of its economic life in 2011. The new equipment would be more efficient than the old equipment, resulting in a 25 percent reduction in both direct material and variable overhead. The savings in direct material would result in an additional one-time decrease in working capital requirements of P2,500, resulting from a reduction in direct material inventories. This working capital reduction would be recognized at the time of equipment acquisition.

. The initial net cash outflows if the company decides to continue making the waste containers is: A. P 956,600 C. P 978,900 B. P 975,500 D. P1,455,613

lxxxvi

The old equipment is fully depreciated and is not included in the fixed overhead. The old equipment from the plant can be sold for a salvage amount of P1,500. Rather than replace the equipment, one of Franzen’s production managers has suggested that the waste containers be purchased. One supplier has quoted a price of P27 per container. This price is P8 less than Franzen’s current manufacturing cost, which is presented below. Direct materials Direct labor Variable overhead Fixed overhead: Supervision Facilities General Total unit cost

lxxxvii

P10 8 6 P2 5 4

. The total after-tax cash outflows, excluding the initial cash outflows, if the new equipment is purchased are: A. P 956,600 C. P2,918,300 B. P2,887,800 (defective) D. P3,279,000 . The present value of the total depreciation shield is: A. P308,920 C. P307,826 B. P313,500 D. P321,303

lxxxviii

.The total relevant after-tax costs to buy the waste containers are: A. P2,829,240 C. P4,243,500 (defective B. P3,039,662 D. P7,074,000

lxxxix

. What is the net present value of the purchase alternative? A. P3,039,662 (defective) C. P2,083,062 B. P2,730,742 D. P2,718,359

11 P35

Franzen uses a plantwide fixed overhead rate in its operations. If the waste containers are purchase outside, the salary and benfits of one supervisor, included in fixed overhead of P45,000 would be eliminated. There would be no other changes in the other cash and noncash items included in fixed overhead except depreciation on the new equipment.

xc

The new equipment will be depreciated according to the following declining amounts: Year Depreciation 2007 P319,968 2008 426,720 2009 142,176 2010 71,136 2011 0

ANSWER EXPLANATIONS

. What is the net present value of the make alternative? A. P2,036,603 C. P2,996,603 B. P3,039,662 D. P2,993,203 (defective)

Franzen is subject to a 40 percent tax rate. Management assumes that all cash flows occur at the end of the year and uses a 12 percent after-tax discount rate. 247

i

.

Answer: B Initial amount of investment Less Cash inflow (decrease in outflow) at period 0: MV of old equipment Tax benefits on loss on sales (20,000 x .4) Net investment

ii

.

iii

.

iv

10. Answer: B

v

vi

160,000 80,000 8,000

88,000 72,000

Answer: D ATCF = Net investment ÷ Payback period ATCF (840,000 ÷ 3.326) 252,555 Net income (252,555 – 140,000) 112,555 Before-tax income (112,555 ÷ 0.60) 187,592 Before-tax savings (187,592 + 140,000) 327.592 The computation of after-tax cash flows, given the amount of investment and internal rate of return or PV of annuity of 1 discounted at IRR is the reverse of the computation of payback period. Remember that the payback method, though a nondiscounted technique, is closely related to internal rate of return because the payback period is exactly the present value of annuity of 1 if they are discounted using the internal rate of return. Answer: A Annual savings on expenses P50,000 Less: Additional depreciation (40,000 – 25,000) 15,000 Additional taxable income 35,000 Additional tax (35,000 x 40%) P14,000 Additional depreciation can be easily calculated by subtracting the book value of the old machine from the cost of new machine and then the difference divided by the useful life (160,000 – 100,000) ÷ 4 = 15,000.

YearSYDStraight LineDifferencePresent Value12,000,0001,200,000800,000 727,28021,600,0001,200,000400,000330,56031,200,0001,200,000 -04 800,0001,200,000(400,000) (273,200)5 400,0001,200,000(800,000) (496,720)Total present value of difference in depreciation287,920Tax Rate40%Present value of net advantage115,168 . Answer: B SYDSLDifferencePresent Value1 150,00090,00060,00053,5682 120,00090,00030,00023,9163 90,00090,000-04 60,00090,000(30,000)(19,066)5 30,00090,000(60,000)(34,046)Total of present values of depreciation24,372Tax rate40%Present value of net advantage 9,749SYD method provides a higher present value on tax benefits because of less amount of tax during year 1 & 2. In year 4 and 5, the use of SYD requires higher taxes but their equivalent present values are lower already. .

Answer: D Annual cost savings Less depreciation Annual income Simple Rate of Return:

(432,000 ÷ 12) 54,000 ÷ 432,000

90,000 36,000 54,000 12.5 %

vii

.

viii

.

ix

. Answer: D The average (accounting) rate of return is determined by dividing the annual after-tax net income by the average cost of the investment, (beginning book value + ending book value)/2. After tax income (P7,200 - (P7,200 x 30%)) P 5,040 Average investment: (P66,000 + 16,000) ÷ 2 P41,000 Accounting rate of return: P5,040/P41,000) 12.3%

Answer: A The useful life of the project can be calculated by using the computational pattern for Accounting Rate of Return: Net investment 106,700 Divide by Depreciation expense CFAT 20,000 Less: Net income (106,700 x 5%) 14,665 5,335 Average life (in years) 7.28 * 10% ARR based on average investment = 5% ARR based on initial investment

Answer: B ARR = Average annual net income ÷ Average Investment Annual after-tax cash flow 40,000 Less Depreciation 20,000 Net Income 20,000 Divide by Average Investment (200,000 + 180,000)/2 190,000 ARR: 10.5% The problem asked for the average accounting rate of return for the first year of asset’s life.

x

.

Answer: A (ATCF – Depreciation) ÷ Initial investment = Accounting Rate of Return Let X = Initial investment (66,000 – 0.10X) ÷ X = 0.12 66,000 - .10X = .12X .22X = 66,000 X = 300,000

xi

.

Answer: A Net Income: = 66,000 - .10X AAR = NI/ Investment .12 = (66,000 - .10X) / X .12X = 66,000 - .10X .22 X = 66,000 X = 300,000

xii

.

Answer: D Net Income (280,000 x 15%) Add back depreciation ATCF

42,000 35,000 77,000

xiii

. Answer: B Payback period = Initial amount of investment ÷ Annual after-tax cash flows P35,000 ÷ P5,000 = 7 years

xiv

.

xv

.

xvi

.

Answer: B Net investment Divide by CFAT (10,000 x 0.7) ÷ (50,000 ÷ 8 x 0.3) Payback period Answer: D Cumulative cash flows end of Year 1 Discounted cash flow for Year 2 Cumulative cash flows, end of Year 2 Break-even time

(450,000) – 254,520 2 + (22,020 ÷ 105,140)

Answer: D Cost of the new machine Salvage value of old machine at period zero Net investment (Outflows) Divide by cash flow after tax Payback period

50,000 8,875 5.6 years (195,480) 173,460 ( 22,020) 2.21 years 400,000 60,000 340,000 90,000 3.78 years

xvii

. Answer: B Cash InflowUnrecovered OutflowOutflows(4,500,000)First year900,000(3,600,000)Second year1,200,000(2,400,000)Third year1,500,000( 900,000)Fourth year 900,0000 Payback Period: At the end of 4 periods, the initial outflows are fully recovered. Note to the CPA Candidates: A modified question for this problem is to compute the Present Value of the net advantage of using sum-of-the-years’ digits of depreciation instead of straight-line method.

xviii

.

xix

.

Answer: C Cash inflowsInvestmentPeriod 0(99,300)Period 1 (75,000 – 25,000) x .6 30,000(69,300)Period 2 ( 30,000 x 1.10) 33,000(36,300)Period 3 (33,000 x 1.10) 36,300 -0-At the end of the third year, investment is fully recovered. The net investment of 99,300 is net of tax benefit, (165,500 x .6) Answer: C Before-tax cash flow = 40,000 + 35,000 Payback period: 300,000 ÷ 75,000

xx

.

xxi

. Answer: C Computation of Cash Flow After-tax CFBT 100,000 x 0.7 Depreciation tax shield 62,500 x 0.3 CFAT Computation of Net Present Value:

75,000 4 years

Answer: C There are two cash flows at time zero: P120,000 outflow and P14,000 inflow. Net cash outflow (120,000 – 14,000) = 106,000

70,000 18,750 88,750

PV of ATCF: 88,750 x 5.747 510,046 PV of After-tax Salvage Value: 20,000 x 0.70 x 0.54 7,560 Total 517,606 Investment 500,000 Net Present Value 17,606 The problem assumed that the salvage value is ignored in the computation of annual depreciation so that the annual cash flows will be greater. The problem did not include among the choices the assumption that salvage value will be deducted from the cost in computing the amount of annual depreciation. xxii

.

Answer: B Annual revenues Less cash operating costs Cash flow before tax Less Depreciation (1M ÷ 5) Income before tax Less income tax (40%) Net income Add back depreciation ATCF PV of ATCF, n=5; k=10% Investment Negative Net Present Value

400,000 104,800 295,200 200,000 95,200 28,080 57,120 200,000 257,120 974,690 1,000,000 ( 25,310)

257,120 x 3.7908

The manner of financing the project is not considered in the analysis of capital investment. Investment must be separate from financing. It is a normally committed error in the application of capital budgeting techniques where financing strategy is considered. The explicit or implicit cost of financing the project is taken care of the discounting process. xxiii

.

xxiv

.

xxv

Answer: A Present value of cash returns: (30,000 x 0.90909) x 5 periods 136,364 Net investment 99,300 Net present value 37,064 Note: Because the constant growth rate and the discount rate are both 10%, the present value for each period is constant.

.

Answer: B Savings (2 workers, each P10,000 for 3 months) 2 x P10,000 x 3 Depreciation (175,000 – 25,000) ÷ 5 years After-tax cash savings: (60,000 x 0.75) + (30,000 x 0.25) Present value of after-tax cash savings (52,500 x 3.60478) Present value of Salvage Value (25,000 x 0.56743) Total Investment Net Present Value Answer: B Computation of net investment: Cash purchase price Less: MV of old machine Tax shield on loss on sale (40,000 x 0.32) Net investment

300,000 80,000 12,800

Annual cash savings before tax (240,000 – 160,000) Additional depreciation (300,000 – 120,000) ÷ 4 Additional taxable income Less Additional tax (35,000 x 0.32) Net income Add back depreciation After-tax cash flow Alternative computation for ATCF: (80,000 x 0.68) + (45,000 x 0.32) Present value of ATCF (68,800 x 3.23972) Investment Net Present Value xxvi

.

Answer: B PV of annual cash receipts PV of salvage value PV of return of working capital Cost of new equipment and timbers

P60,000 P30,000 P52,500 P189,250 14,186 203,436 175,000 P 28,436

1,200,000 x 2.58872 650,000 x 0.48225 1,000,000 x 0.48225

92,800 207,200 80,000 45,000 35,000 11,200 23,800 45,000 68,800 68,800 222,893 207,200 15,693 3,106,463 313,462 482,250 (2,750,000)

Working capital PV of cost of construction of road Negative net present value xxvii

.

400,000 x .5787

(1,000,000) ( 231,480) (79,303)

Answer: B PeriodNominal Cash SavingsPV FactorPresent Value132,000 0.8779028,070.08232,000 x 1.0533,600 0.7694725,854.19332,000 x 1.05235,280 0.6749723,812.94432,000 x 1.05337,044 0.5920821,933.01Total99,670.22Investment80,000.00NPV19,670.22Note that all the annual cash inflows are adjusted by one period.

xxviii

. Answer: B The solution used total analysis approach in computing present value. Retain the Old Machine: Present value of annual cash outlay CFAT (300,000 x P0.38) + P21,000 = P135,000 PVCFAT (135,000 x 3.6847) Present value of salvage value (7,000 x 0.41044) Total

P497,435 ( 2,873) P494,562

Buy New machine: Present Value of Annual cash outlay CFAT (300,000 x P0.29) + P11,000 = P98,000 PVCFAT P98,000 x 3.6847) Salvage value of new machine, end of 6 years(P20,000 x 0.41044) Investment in new machine (120,000 – 40,000) Total

P361,100 ( 8,209) 80,000 P432,891

xxix

. Answer: B The purpose of profitability index is to compare two projects’ profitability by reducing the present value per 1 peso of investment. Therefore, the ratio of 4.35526 @ 10% to 4.11141 @ 12% indicated the profitability index. Profitability index: 4.35526/4.11141 = 1.06

xxx

.

Answer: B PV of annuity of 1 at IRR ∑(1 ÷ 1.12386)5 PV of annuity of 1 at MCC ∑(1 ÷ 1.11055)5 After-tax cash flows 10,000 ÷ (3.69079 – 3.57057) Investment: 83,180.84 x 3.57057 Profitability index (297,000 + 10,000) ÷ 297,000 A shorter calculation of the Profitability Index can be made by: 3.69079 ÷ 3.57057 = 1.034

3.57057 3.69079 83,180.84 297,000 1.034

xxxi

. Answer: D In discounting the annual cash inflow by the IRR, the NPV = P0 The net present value of ZERO is 14% and 16%. For better time management, the candidate is expected not to do detailed calculation of finding out the exact rate. The use of interpolation indicated that the IRR is 15.3%: Discount RateNet Present Value0.141,197IRR00.16-708 (0.14 – IRR) ÷ (0.14 – 0.16) = 1,197 ÷ ( 1,197 + 708) (0.14 – IRR) ÷ -.02 = 1,197 ÷ 1905 (0.14 – IRR) ÷ - .02 = 0.628 (0.14 – IRR) = 0.628 x -0.02 0.14 – IRR = 0. 013 IRR = 0.153 or 15.30% Note: Since at the IRR, NPV is zero, the answer can only be between 14% & 16%, since only one of the choices, satisfy the criteria, the answer is (D).

xxxii

. Answer: B The payback period that corresponds to the project’s internal rate of return of 12 percent is 4.968. Therefore, the amount of investment must equal the product of the payback period and the net cash flows: Investment: (4.968 x 20,000) = P99,360

xxxiii

. Answer: D The amount of investment: the PV of annuity at IRR 4.355 x 6,000 = 26,130

xxxiv

. Answer: C Present value of cash inflows equals amount of investment at 10% IRR. P20,000 x 3.791 = P75,820

xxxv

xxxvi

.

.

Answer: A ATCF: Depreciation Net income: Before-tax income: Fixed costs Contribution margin: Unit sales

P1,500,000/3.60472

416,121 300,000 116,121 193,535 500,000 693,535 17,338

416,121 – 300,000 116,121/0.60 193,535 + 500,000 693,535 ÷ (100 - 60)

Answer: B Contribution margin (per No. 23) Divide by sales volume Contribution margin per unit Add variable cost per unit Selling price per unit

693,535 ÷ 20,000 P34.68 60.00 P94.68

Alternative Solution: Cash inflow before tax based on present price: (20,000 x 40) – 200,000 After-tax cash inflow (600,000 x 0.6) + (300,000 x 0.4) Present value of ATCF (480,000 x 3.60478) Investment Net present value (present price) Annual excess ATCF due to excess price (230,294 ÷ 3.60478) Before-tax excess cash inflow (63,885 ÷ 0.6) Excess selling price: 106,475 ÷ 20,000 Reduced selling price to achieve IRR of 12% (100 – 5.32) xxxvii

.

Answer: C Annual after-tax cash flow Depreciation Net income Income before tax Depreciation Cash savings before tax:

500,000/5.6502 500,000/10 38,492/0.6 64,154 + 50,000

600,000 480,000 1,730,294 1,500,000 230,294 63,885 106,475 5.32 94.68 88,492 50,000 38,492 64,154 50,000 114,154

xxxviii

. Answer: A The amount of annual cash flows can be solved by equation: NPV = PV of annual CF – Investment 1,750 = 2.4771CF – 2.4018CF 1,750 = 0.1753CF CF = 9,980

xxxix

xl

xli

xlii

.

.

.

.

Answer: A Investment Less Present value of salvage value Present value of Annual Cash Inflows Minimum Annual Cash Flows

(12,000 x 0.3855) (115,374 ÷ 6.1446)

Answer: B Present value of annual cash flows at IRR Investment Difference Annual increase in cash flows

(81,000 x 4.639) 81,000 x 4.344 23,895/4.344

Answer: A Investment (Total of present value @ IRR of 12%) Less PV, year 1 & 2 (16,074 + 17,534) PV of the 3rd cash flow After-tax cash flow, third year 16,392/0.712

120,000 4,626 115,374 18,776 375,759 351,864 23,895 5,501 50,000 33,608 16,392 23,022

Answer: B The net present value = PV of excess salvage value less PV of decrease in after-tax cash flow Let X = the excess salvage value 7,003 = 0.56743X – [3.60478 x (0.2X * 0.4) 7,003 = 0.56743X – 0.2883824X 7,003 = 0.2790476X X = 25,096 Required salvage value: 50,000 – 25,096 = 24,904

xliii

xliv

.

Answer: B Cost of equipment Less PV of tangible benefits PV of annual intangible benefits Amount of annual intangible benefits

100,000 x 5.01877 248,123/5.01877

750,000 501,877 248,123 49,440

. Answer: B To be acceptable, the project should yield a net present value of zero. The negative net present value must be offset by the present value of annual intangible benefits. Present value of intangible benefits P184,350 PV of annuity of 1 at 10% for 10 years ÷ 6.145 Annual net intangible benefits P30,000

xlv

. Answer: A The indifference rate (crossover or fisher rate) refers to the rate at which the net present values of the 2 alternatives are indifferent or equal. The easier test of the rate is to look for IRR (using trial and error technique) of the investment difference. Difference 80,000 – 48,000 35,225 PV inflows ∑(3,200 ÷ 1.1264)6 (12,922) PV inflows ∑(15,200 ÷ 1.1264)10-6 (22,303) Difference NIL Alternative Solution: Project XProject YPV of after-tax cash flows ∑(12,000 ÷ 1.1264)6

48,455 ∑(15,200 ÷ 1.1264)1083,680Investment48,00083,225Net Present Value

455

455

xlvi

. Answer: B The determination of the indifference point, which is 10%, for the two projects can be made through the use of trial and error estimation. Machine 1Machine 2PV of Difference in ATCF Year 1 155,000 ÷ 1.10 140,909.10(140,909.10) Year 2 (110,000 ÷ 1.10)2( 90,909.10) 90,909.10Net difference 50,000.00( 50,000.00)Difference in investment( 50,000.00) 50,000.00NPV NIL NIL xlvii . Answer: C 15% Discount Rate Machine 1Machine 2PV of Difference in ATCF Year 1 155,000 x 0.86957 134,783.35(134,783.35) Year 2 110,000 x 0.75614( 83,175.40) 83,175.40Net difference 51,607.95( 51,607.95)Difference in investment( 50,000.00) 50,000.00NPV 1,607.95( 1,607.95) At 15 percent discount rate, Machine 1 is more acceptable. 8% Discount Rate Machine 1Machine 2PV of Difference in ATCF Year 1 155,000 x 0.92593 143,519.15(143,519.15) Year 2 110,000 x 0.85734( 94,307.40) 94,307.40Net difference 49,211.75( 49,211.75)Difference in investment( 50,000.00) 50,000.00NPV ( 788.25) 788.25 At 8 percent discount rate, Machine 2 is more acceptable. xlviii

. Answer: C Cost of Investment: Invoice price Installation cost Freight charge Total investment

950,000 24,200 800 975,000

Annual Cash Flow: Number of procedures: Contribution margin per procedures: Total annual cash flow: Cash payback period: xlix

l

.

.

Answer: B Present value of cash flow Present value of salvage value Total Capital investment Net present value Answer: A Average investment: Annual depreciation: Annual net income: Average annual Rate of Return:

(52 x 5) (P800 – P10 – P40) (260 x P750) (975,000 ÷ 195,000)

(195,000 x 6.418) (55,000 x 0.42241)

(975,000 + 55,000) ÷ 2 (975,000 – 55,000) ÷ 10 195,000 – 92,000 P103,000  P515,000

260 P750 P195,000 5 years P1,251,510 23,233 P1,274,743 975,000 P 299,743 515,000 92,000 103,000 20%

li

lii

liii

.

.

.

liv

.

lv

.

lvi

Answer: A Contribution margin: Less Fixed costs Cash flow before tax Less: Depreciation Income before tax Less: Income tax Net income Add back: Depreciation After-tax Cash Flow Answer: C PV of After-tax Cash Flows Cost of investment Net Present Value

300,000 x (75 – 50) (6,000,000 ÷ 4) (1,500,000 x 0.4)

(2,400,000 x 2.9287)

Answer: A Annual excess present value Excess cash before tax Maximum number of units as decrease

(1,028,000 ÷ 2.9287) (351,000 ÷ 0.6) (585,000 ÷ 15)

7,500,000 4,500,000 3,000,000 1,500,000 1,500,000 600,000 900,000 1,500,000 2,400,000 7,028,900 6,000,000 1,028,900 P351,000 P585,000 39,000

Answer: A Average Annual net income: (100,000 + 40,000 + 20,000 + 10,000 + 10,000) ÷ 5 = 36,000 Divide by average investment (400,000 ÷ 2) 200,000 Accounting rate of return 18% Accounting rate of return or unadjusted rate of return computes the profitability of the project in term of accrual profit. Net profit under accrual method considers depreciation, a substantial amount that understates the average profit. This understatement of amount that is used in the computation necessarily requires that preferably, average investment should be used, instead of the initial investment, in the determination of accounting rate of return. Answer: B Cash FlowPV FactorPV of annual net cash flows:180,0000.909163,620120,0000.826 99,120100,0000.751 75,10090,0000.683 61,47090,0000.621 55,890Total455,200Amount of investment400,000Net Present Value 55,200 . Answer: C Present Value Index (Profitability Index) Present Value of ATCF ÷ Net Investment (455,200 ÷ 400,000) = 1.14 The present value index computes net present value in terms of P1 investment. Therefore, the index of 1.14 means the net present value per P1 of investment is P0.14. This concept makes the present value index better than the net present value technique because the index indicates which one is the most profitable on a per P1 investment.

lvii

. Answer: D Cash InflowUnrecovered InvestmentPeriod 0 Outflows(400,000)Period 1180,000(220,000)Period 2120,000(100,000)Period 3100,000Zero The total outflows are fully recovered by the end of period 3. The analyst should be careful in computing the payback period when the project has uneven cash inflows. The common error in handling uneven cash flows is using the average cash flows instead of reducing the unrecovered outflows.

lviii

.

Answer: D Payback period: Investment ÷ Net Annual Cash Inflow P200,000 ÷ P50,000 = 4 years

lix

.

Answer: D Present value of Net Cash Inflow (71,000 X 4.355) Investment Net Present value

309,205 300,000 9.205

lx

. Answer: B Average Investment: (200,000 ÷ 2) = 100,000 Accounting Rate of Return = Net Income ÷ Average Investment (10,000 ÷ 100,000) = 10 percent

lxi

. Answer: B The payback for PA is 4.225. This is closest to the present value of annuity of 1 discounted at 11 percent for 6 periods which is 4.231.

lxii

.

Answer: A Annual depreciation: (P50,000 ÷ 8) Annual tax shield: (P6,250 x 0.3)

P6,250 P1,875

lxiii

.

lxiv

.

Answer: B Payback period: (P50,000 ÷ P8,875) = 5.6 years

lxv

.

Answer: C Present value of annual ATCF (P8,875 x 5.747) Present value of after-tax salvage value (P1,400 x 0.54) Total Investment Net present value

Answer: C Before-tax cash inflow P10,000 Less depreciation 6,250 Income before tax 3,750 Less income tax (3,750 x 0.3) 1,125 Net income 2,625 Add back depreciation 6,250 After-tax cash inflow P 8,875 A quicker calculation of after tax cash flow can be made by adding the tax shield to after-tax cash inflow without any tax benefit on depreciation. (P10,000 × .70) + P1,875 = P8,875

P51,000 756 51,756 50,000 P 1,756

lxvi

. Answer: C At the discount rate of 8 percent, there is a net present value of P1,756. Therefore, the IRR is higher than 8 percent. Using trial and error approach, the first try should use 9 percent. If the present value of the inflows exceeds P50,000, then the IRR is lower than 9 percent, otherwise it should be 9.5 percent. Using 9.0 percent in discounting the inflows, there is a net present value of P(174); therefore the IRR is slightly lower than but very close to 9.0 percent. (P8,875 x 5.535) + (P1,400 x 0.5019) – P50,000 = P(174)

lxvii

.

lxviii

lxix

lxx

.

.

.

Answer: B Additional contribution margin: Small 6,000 x 5.40 Medium 15,000 x 6.50 Large 9,000 x 7.90 Total Less Cash Fixed Expenses: Rent Salaries Utilities Insurance, etc. Annual Cash Inflows Answer: B Additional rental income Additional cash flow, snack bar Total Less Cash Fixed Expenses: Rent Salaries Utilities Insurance, etc. Annual Cash Inflow Answer: A PV of annual cash inflow PV of salvage value PV of working capital return Total Investment: Remodeling cost Working capital Net Present Value

32,400 97,500 71,100 201,000 18,000 54,000 13,200 7,800

93,000 108,000 130,000 15,000 145,000

48,000 17,000 5,400 9,600

(108,000 x 5.575) (70,000 x 0.108) (7,500 x 0.108)

Answer: B PV of annual cash inflow (65,000 x 5.575) PV of salvage value PV of working capital return

80,000 65,000 602,100 3,240 810 606,150

550,000 7,500

557,500 48,650 362,375 1,296 432

Total Investment: Remodeling cost Working capital Net Present Value lxxi

.

364,103 290,000 4,000

Answer: A Rental income 21,000 x 5 Additional cash inflow, snack bar Total Less fixed expenses Annual cash inflow PV of annual cash inflow PV of salvage value PV of working capital return Total Investment Negative Net Present Value

294,000 70,103 105,000 13,000 118,000 80,000 38,000

(38,000 x 5,575)

211,850 1,296 432 213,578 294,000 ( 80,422)

lxxii

. Answer: D The annual cost of advertising can be easily calculated by dividing the net present value of alternative 2, at 16% by the present value of annuity of 1. 70,103 ÷ 5,575 = 12,574.53

lxxiii

.

lxxiv

.

Answer: A Annual revenues Variable expenses Contribution margin Fixed expenses Advertising Salaries Utilities Insurance Annual cash income Less Depreciation Annual Income

300,000 60,000 240,000 40,000 110,000 5,200 800 420,000 x 0.90 ÷ 12

Answer: A Current operating costs – old machine Deduct Operating costs – Machine B Annual salary of operator Annual maintenance cost Annual cash savings

lxxv

. Answer: A Simple Rate of Return = Net Income ÷ Initial Investment 52,500 ÷ 420,000 = 12.50 %

lxxvi

.

lxxvii

234000 ÷ 13 years 38,250 ÷ 225,000

78,000 16,350 5,400

21,750 56,250

56,250 18,000 38,250 17 %

. Answer: C Payback period = Initial Investment ÷ Annual Cash Inflow 420,000 ÷ 84,000 = 5 years

lxxviii

lxxix

Answer: B Savings Less Depreciation Annual income Simple Annual Return

156,000 84,000 31,500 52,500

.

Answer: A 225,000 ÷ 56,250 = 4 years

.

Answer: C Purchase price of new tools Add increase in working capital Total Deduct Salvage value of the old tools Net investment

2,500,000 60,000 2,560,000 45,000 2,528,500

lxxx

.

lxxxi

.

lxxxii

.

lxxxiii

lxxxiv

lxxxv

lxxxvi

.

.

.

.

Answer: C Purchase price of valve stem 80,000 x 20 Cost to make: Direct materials Direct labor Variable overhead Decrease in directs labor and variable costs Cost savings

80,000 x 4.50 80,000 x 3.90 80,000 x 1.50 80,000 x 1.60

360,000 312,000 120,000 (128,000)

664,000 936,000

Answer: A PV of annual depreciation PeriodDepreciationPV FactorPresent ValueYear 1 832,5000.89286743,305.95 2 112,5000.79719886,873.88 3 370,0000.71178263,358.60 4 185,0000.63552117,571.20Total2,011,109.63Tax rate0.30PV of tax benefits from depreciation603,332.89 Answer: C After tax salvage value 100,000 x .7 70,000 PV of 1 end of 4 periods 0.63552 PV of after – tax salvage value 44,486.4 Answer: C PV of after cash savings 936,000 x .7 x 3.03735 PV of tax benefits from depreciation PV of after tax salvage value PV of working capital return 60,000 x 0.63552 Investment Net present value

1990072 603,333 44,486 38,131 (2528,500) 147,522

Answer: A PV of tax benefits, declining - balance PV of tax benefits, straight-line method 2,500,000  4 x .3 x 3.03735 Net advantage Answer: A Invoice price of new equipment (945,000 x 0.98) Freight Installation cost Total Less: Salvage value of old equipment (0.6 x 1,500) Reduction in working capital Net initial outflows Answer: B Total variable costs Avoidable fixed costs Total After-tax Cash outflows Operating expenses Depreciation After-tax salvage value of new equipment Net outflows *Variable cost per unit Direct material (10.00 x 0.75) Direct labor Variable overhead (6.00 x 0.75) Total

lxxxvii

1,600,000

603,333 569,503 33,830 P926,100 11,000 22,900 960,000

900 2,500

3,400 P956,600

(262,000 units x P20*) (P45,000 x 5 years)

P5,240,000 225,000 5,465,000

(5,465,000 x 0.6) (960,000 x 0.4) (12,000 x 0.60)

P3,279,000 ( 384,000) ( 7,200) P2,887,800 P 7.50 8.00 4.50 P20.00

. Answer: A The present value of the tax shield based on declining-depreciation is: YearDepreciationTax Shield (40%)PV FactorPV of Tax Shield2007P319,968P127,9870.893P114,2922008 426,720 170,6880.797 136,0382009 142,176 56,8700.712 40,4922010 71,136 28,4550.636 18,098TotalP308,920 lxxxviii . Answer: C Purchase Cost Year ATCF200750,000 x 27 x 0.6 810,000200850,000 x 27 x 0.6 810,000200952,000 x 27 x 0.6 842,400201055,000 x 27 x 0.6 891,000201155,000 x 27 x 0.6 891,0002006(1,500 x 0.6) ( 900) Total 4,243,500 lxxxix . Answer: A Present value of after-tax cash flows

2007 2008 2009 2010 2011 Salvage value of old equipment Net present value

(810,000 x 0.893) (810,000 x 0.797) (842,400 x 0.712( (891,000 x 0.636) (891,000 x 0.567) (1,500 x 0.60)

P 723,330 645,570 599,789 566,676 505,197 (900) P3,039,662

xc

. Answer: D CFBTCFATPV FactorPVCFAT2006Initial outflow(P956,600)2007(50,000 x 20) + 45,000 (1,045,000 x 0.6) - (319,968 x 0.4)1,045,000 499,013 0.893 445,6192008(1,045,000 x 0.6) – (426,720 x 0.4)456,3120.797363,6812009(52,000 x 20) + 45,000 (1,085,000 x 0.6) – (142,176 x 0.4)1,085,000 594.130 0.712 423,0212010(55,000 x 20) + 45,000 (1,145,000 x 0.6) – (71,136)1,145,000 658,546 0.636 418,8352011(55,000 x 20) + 45,000 (1,145,000 x 0.6)1,145,000 687,000 0.567 385,447Salvage value (12,000 x 0.6)7,200P2,993,203

View more...

Comments

Copyright ©2017 KUPDF Inc.
SUPPORT KUPDF